Jack, Davis, Faiza, and Marry want to take a picture together. In how many ways can they are arranged?

Answers

Answer 1

Answer:

16

Step-by-step explanation:


Related Questions

Please answer correctly !!!! Will mark brainliest !!!!!!!!!!!!

Answers

Answer:

[tex]x^2+10x+24[/tex]

Step-by-step explanation:

So just multiply the sides:

[tex]x^2+6x+4x+24[/tex]

Which is:

[tex]x^2+10x+24[/tex]

What is the shape of the cross-section formed when a cylinder intersects a
plane as shown in the drawing?

Answers

Answer:

circle,

Step-by-step explanation:

disc in the middle vertically in this case is equal to the bases

The intersection is called an Oval. Hence the correct option is A. An oval in mathematics is a shape just like a circle but with an elongated outline like the shape of an egg.

What is a Cross-section?

A cross-section is a surface, an area that is created or exposed by executing a straight cut across or through a shape.

Cross-sections in technical drawings are used for depicting the internal view of an object that is three-dimensional.

Learn more about Cross-Sections are:

https://brainly.com/question/10511133

The circumference of the earth is given. 


Circumference of earth: 24,901 miles


What is the diameter of earth? Round your answer to the nearest tenth. Use 3.14 for π.


Answers

Answer:

7930.3 miles = d

Step-by-step explanation:

The circumference equals

C = pi *d

24901 = 3.14 d

Divide each side by 3.14

24901 / 3.14 = d

7930.254777 = d

Rounding to the nearest tenth

7930.3 =d

HELPPPPPP ITS ABOUT EQUATIONSSSS HELPPPPP Explanation needed HELPPPPP PLEASEE THIS IS THE LAST QUESTION LEFTTTTTTT

Answers

Answer:

C. the average total cost for the first month of a gym membership

Step-by-step explanation:

x=1 is for month 1,

the value of y includes 1-month fee and one off payment, so this is the average total for the first month of membership

Answer:

It is the average total cost for the first month of a gym membership

Step-by-step explanation:

y = 34.99x+49

The 49 is the cost to join the gym and the 34.99 is the monthly cost

Let x =1 which is the cost after one month

It includes the cost to join and the 1st month membership

It is the average total cost for the first month of a gym membership

8. Where will the hour hand of a clock stop if it starts:
a.
from 7 and turns through 1 right angle?
b. from 11 and turns through 3 right angles

can you plz say me the answer​

Answers

Answer:

a. 11

b. 9

Step-by-step explanation:

thats the answer

please solve y = 3x - 1​

Answers

Step-by-step explanation:

i think question is not complete.

k (t) = 10t - 19
k(-7) =

Answers

Answer:

k(-7) = - 89

Step-by-step explanation:

k (t) = 10t - 19

Let t = -7

k(-7) =10 * -7 - 19

        =-70-19

        -89

Answer:

[tex]k (-7) = -89[/tex]

Step-by-step explanation:

[tex]k (t) = 10t - 19[/tex]

[tex]k (-7) = 10(-7) - 19[/tex]

[tex]k (-7) = -70 - 19[/tex]

[tex]k (-7) = -89[/tex]

En un programa de televisión se hacen 30 preguntas. Por cada respuesta correcta se suman 8 puntos, por cada respuesta errónea se restan 5 puntos y por aquellas preguntas que no se contesten no se suman ni se restan puntos. Si un participantes obtuvo 13 puntos, ¿Cuantas respuestas erróneas pudo tener?

Answers

Answer:

7 wrong answers.

Step-by-step explanation:

We know that when you answer a question well, you earn 8 points and a bad one loses 5 points, this means that when you answer 1 good and 1 bad, there is a total of 3 points (8-5).

This means that when answering 5 good and 5 bad, in total it would be 3 * 5 points, that is to say 15 points, that is, it goes through 2 points. To subtract 2 points, you would have to answer 1 good and 2 bad, (8 - 2 * 5), which turns out to be -2 points.

In total there would be 6 questions right and 7 questions wrong, like this:

8 * 6 + 5 * 7 = 13

13 points, in 13 questions (6 correct + 7 incorrect)

The rest of the questions were not answered so that the score does not go up or down.

Which means you got 7 wrong answers.

A man divides his 360 cattle between his son in the ratio 7: 6: 5. Find the smallest share​

Answers

360/18=20

20*7=140

20*6=120

20*5=100

Answer=140:120:100

Answer: smallest share is 100

Step-by-step explanation:

7:6:5

Add the numbers so 7+6+5=18

Divide 360 by 18 which equals 20

Multiply each ratio by 20 so...

7×20:6×20:5×20

140:120:100

(6) Work out
5 1
6 12

Answers

Answer:

=123

Step-by-step explanation:

(3x5−2x4−5)−(2x4+x2−10) Subtract the two polynomials

Answers

Answer:

3x^5-4x^4-x^2+5

Step-by-step explanation:

(3x^5−2x^4−5)−(2x^4+x^2−10)

Distribute the minus sign

(3x^5−2x^4−5)−2x^4-x^2+10

Combine like terms

3x^5-4x^4-x^2+5

Hello!

Answer:

[tex]\boxed{ \bf 3x^5~-~4x^4~-~x^2~+~5}[/tex]

__________________________________Explanation:

(3[tex]x^{5}[/tex] - 2[tex]x^{4}[/tex] - 5) - (2[tex]x^{4}[/tex] + x² - 10)

Drop the brackets:

3[tex]x^{5}[/tex] - 2[tex]x^{4}[/tex] - 5 - 2[tex]x^{4}[/tex] - x² + 10

Combine Like Terms:

3[tex]x^{5}[/tex] - 2[tex]x^{4}[/tex] - 2[tex]x^{4}[/tex] - x² - 5 + 10

3[tex]x^{5}[/tex] - 4[tex]x^{4}[/tex] - x² + 5

Can someone please help

Answers

Answer:

The numbers to input are 2, 2 , 7

Step-by-step explanation:

252 = 2² * 3² * 7

So the numbers to input are 2, 2 , 7

John has two jobs and earns a total of $2,345 per month. What percent of his gross income does John receive from his second job where he makes $609.70 a month?

Answers

Answer:

10000000.9

Step-by-step explanation:

How many digits will be in the quotient?
39 4,641

Answers

Answer:

the answer is 119 so three digits

Step-by-step explanation:

Answer:

3 digits

Step-by-step explanation:

4641/39=119

Brass contains copper and zinc in the ratio 2:1. How much ZINC is there in 750 g of brass?

Answers

Answer:

250

Step-by-step explanation:

copper: zinc : total

2              1      2+1 =3

We have 750 brass

750/3 = 250

Multiply each by 250

copper: zinc : total

2*250     250  3*250

500        250       750

What the answer to this

Answers

Answer:

I think it might be a I don't know for sure though I just need some more points so I can ask a question myself sorry if this didn't help

Identify the like terms in the expression.

Answers

Answer:D

The answer is D.

Step-by-step explanation:

Like terms are only related to the variable at the end, and the variable also has to have the same exponent or it is not a like term.

pleass more math help

Answers

Can’t answer this question

The line plot shows the result of a survey asking students how many hours they spent reading last week. How many students spent 5 or more hours reading?

Answers

Answer:

15 total students

Step-by-step explanation:

Based on the line plot picture that is attached below it can be calculated that a total of 15 students spent 5 or more hours last week reading. This can be easily calculated by adding all the x's on the marks of 5 hours or more. With both the boys and girl students combined the tally was the following

5 hours : 4 students

6 hours : 1 student

7 hours : 3 students

8 hours : 3 students

9 hours : 0 students

10 hours: 2 students

11 hours : 2 students

12 hours : 0 students

adding up to 15 total students

The lateral area of a right cone which has a base diameter of 4 units and a height of 10 units is:

Answers

Answer:

≈64.08

Step-by-step explanation:

Lateral Area=πr[tex]\sqrt{h^{2} +r^{2} }[/tex]

Radius is 1/2 of the diameter so r=2

Height=10

Substitute:

: [tex]3.141(2)\sqrt{10^{2}+2^{2} } \\6.282\sqrt{100+4} \\6.282\sqrt{104} \\ 64.066.282*10.2\\ 64.076\\\\[/tex]

64.076 rounded = 64.08


Before graduating this year, a senior homeroom was given a survey. Of those surveyed, 24% felt they learned better at home. Of this group, 80% said they plan on taking an online course in college. Of the students who felt they did not learn better at home, 40% said they plan on taking an online course in college

Part A
What is the probability a person who does not plan on taking an online course felt they learned better at home?

A : 2/21
B : 24/125
C : 38/125
D : 19/31
E :None of these

Part B
What is the probability a person who does plan on taking an online course felt they did not learn better at home?

A : 2/21
B : 24/125
C : 38/125
D : 19/31
E : None of these

Answers

Answer:

(A) The correct option is (A).

(B) The correct option is (E).

Step-by-step explanation:

The events can be defined as follows:

X =  students felt they learned better at home

Y = students plan on taking an online course in college

The information provided is:

P (X) = 0.24

P (Y|X) = 0.80

P (Y|X') = 0.40

[tex]P(Y'|X)=1-P(Y|X)\\=1-0.80\\=0.20[/tex]

[tex]P(Y'|X')=1-P(Y|X')\\=1-0.40\\=0.60[/tex]

The Bayes' theorem states that the conditional probability of an event E[tex]_{i}[/tex] given that another event A has already occurred is:

[tex]P(E_{i}|A)=\frac{P(A|E_{i})P(E_{i})}{\sum {P(A|E_{i})P(E_{i})}}[/tex]

(A)

Compute the probability a person who does not plan on taking an online course felt they learned better at home as follows:

Use the Bayes' theorem.

[tex]P(X|Y')=\frac{P(Y'|X)P(X)}{P(Y'|X)P(X)+P(Y'|X')P(X')}[/tex]

              [tex]=\frac{0.20\times 0.24}{(0.20\times 0.24)+(0.60\times 0.76)}\\\\=0.09524\\\\\approx 0.095[/tex]

Thus, the probability a person who does not plan on taking an online course felt they learned better at home is 0.095 or 2/21.

(B)

Compute the probability a person who does plan on taking an online course felt they did not learn better at home as follows:

[tex]P(X'|Y')=1-P(X|Y')\\=1-0.095\\=0.905[/tex]

Thus, the probability a person who does plan on taking an online course felt they did not learn better at home is 0.905.

You are going to use an incline plane to lift a heavy object to the top of a shelving unit with a height of 5 ft. The base of the incline plane is 15 ft from the shelving unit. What is the length of the incline​ plane?

Answers

Answer:

15.8 ft

Step-by-step explanation:

The inclined plane, the base of and the shelving unit form the shape of a right angled triangle.

The hypotenuse is the length of the inclined plane, h.

The base of the triangle is 15 ft.

The height of the triangle is 5 ft.

To find the hypotenuse, h, we have to use Pythagoras rule:

[tex]h^2 = a^2 + b^2[/tex]

where a = height of the triangle

b = base of the triangle

Therefore:

[tex]h^2 = 5^2 + 15^2\\\\h^2 = 25 + 225 = 250\\\\h = \sqrt{250}\\ \\h = 15.8 ft[/tex]

The inclined plane is 15.8 ft long.

Which angle in ADEF has the largest measure?

Answers

Answer:

F is the largest angle

Step-by-step explanation:

The largest angle is opposite the largest side.  The smallest angle is opposite the smallest side.

The largest side is 4 so the largest angle is F

Please answer correctly !!!!!!! Will mark brainliest !!!!!!!!!!

Answers

Answer:

9 years.

Step-by-step explanation:

Make x the number of years passed.

[tex]\frac{39+x}{3+x}=4[/tex]

12 + 4x = 39 + x

27 = 3x

x = 9

Please help me!!!!!!

Answers

Answer:

C

Step-by-step explanation:

The addition of the x^5 term makes it non proportional

Answer:

C.

Step-by-step explanation:

A. y = (6x + 3) - 3= 6x

B. y = - 15x

D. = - 1/3 x

All of these are proportional because they have general formula y = kx.

Find the value of a . A.18 B.21 C.20 D.17

Answers

Answer:

a =18

Step-by-step explanation:

The two angles are vertical angles and vertical angles are equal

6a +11 = 2a+83

Subtract 2a from each side

6a-2a +11 = 2a-2a+83

4a +11 =83

Subtract 11 from each side

4a +11 -11 = 83-11

4a = 72

divide each side by 4

4a/4 = 72/4

a =18

The price of a computer was decreased by 7% to £500. What was the price before the decrease? Give your answer to the nearest penny.

Answers

Answer:

£537.63

Step-by-step explanation:

Let the original price of the computer be x.

The price of a computer was decreased by 7% to £500. This implies that:

x - 7/100 * x = 500

x - 0.07x = 500

0.93x = 500

x = 500/0.93 = £537.63

The price before the decrease was £537.63.

What is the slope of the graph? slope = -1/3 slope = -3 slope = 3 slope = 1/3

Answers

Answer:

The slope is -3

Step-by-step explanation:

This is so because the line in pointing down, leading to the fact that the slope is negative, and rise over run is 3/1......

Therefore, the slope of the line is -3

To find the slope of the line, first start with a point on the graph.

Let's use the point (0,3) which we call point A.

The other point we will call point B.

Now remember that slope can be found using the ratio rise/run

between any two points that are on that line.

To get from point A to point B along this line, we must

first go down 3 units so we say that our rise is -3.

From there we move 1 unit to the right so our run is 1.

So our slope or rise/run is -3/1 which reduces to -3.

Jasper decided to save $100 at the end of each month for a year and deposit it in a bank account that earns an annual interest rate of 0.3%, compounded monthly. Use the formula for an annuity, F, to determine how much money will be in the account at the end of the 6th month, rounding your answer to the nearest penny.
Note: Your interest rate must be converted to a decimal

Answers

Answer: 600.38

Step-by-step explanation:

just put in the answer you lazy

what is the greatest common factor for 9m+ 27m²

Answers

Answer:

9m

Step-by-step explanation:

Both of them are divisible by 9 and 1 m

When you factor and pull out 9m, you will get

9m (1 + 3m)

the greatest common factor for 9m and 27 m is 9
Other Questions
Identify the phase of the design process illustrated in the following scenario, and justify its importance. Kristin has recently designed a hydraulic robot. She is about to visit the site where the robot is currently assembling fragrance bottles; she wants to ensure that it is working correctly and meeting the needs of her client. Write this equation in slope-intercept form. y - 3 = 5(x +3) what was not reaffirmed by the Council of Trent? Which is the solution to the linear equation? 2/3 x-1/2=1/3+5/6x 10.) A study recorded the reactions of 186 polarbears as they were approached by a tundrabuggy. Some bears did not appear to respond,while others responded by sitting, standing,walking away, or running away. There were 94more bears that did not respond than didrespond. How many bears responded and howmany bears did not respond? what is the relationship between reading long-form texts and the development of certain skills? Initially, how did Jing Mei feel about becoming a prodigy? Given the functions f(x) = 7x + 13 and g(x) = x + 2, which of the following functions represents f[g(x)] correctly? Animal populations are not capable of unrestricted growth because of limited habitat and food supplies. Under such conditions the population follows a logistic growth model:P(t) = d1 + kectwhere c, d, and k are positive constants. For a certain fish population in a small pond d = 1400, k = 13, c = 0.2, and t is measured in years. The fish were introduced into the pond at time t = 0.(a) How many fish were originally put in the pond?Correct: Your answer is correct. fish(b) Find the population after 10, 20, and 30 years. (Round your answers to the nearest whole number.)10 years Fish 20 years fish30 years fish If x = 49, what is the value of x(x - 6)? In circle Qwith the measure of minor arc PR= 70, find m/PSR. Simplify3m + 2n - (m + n) Evaluate the factors that shaped U.S. foreign policy in the Middle East during George H.W. Bushs administration A particular group of men have heights with a mean of 181 cm and a standard deviation of 6 cm. Earl had a height of 196 cm. a. What is the positive difference between Earl's height and the mean? b. How many standard deviations is that [the difference found in part (a)]? c. Convert Earl's height to a z score. d. If we consider "usual" heights to be those that convert to z scores between minus2 and 2, is Earl's height usual or unusual? The profit function for the first version of the device was very similar to the profit function for the new version. As a matter of fact, the profit function for the first version is a transformation of the profit function for the new version. For the value x = 40, the original profit function is half the size of the new profit function. Write two function transformations in terms of P(x) that could represent the original profit function. What is the conjugate 8-9 3. In her biography of Plath, Bitter Fame, the poet Anne Stevenson says that this poempenetrates "the furthest reaches of disdain and rage... bereft of all 'normal human feelings."What do you think Stevenson means? Does anything in the poem strike you as particularlychilling? How does the word choice in stanza 1 contribute to the tone at the beginning of the poem?ON WINNING THE COLORING CONTEST IN SECOND GRADE What is the length of jk 1/3(2x-8)=4 algebraic properties